I want to find largest eigenvalue using power iteration. Is it correct?












1












$begingroup$


$$A = begin{pmatrix}4 &–1 & 0 \-1& 4& -1\ 0 &-1& 4 end{pmatrix} $$ $X_0 = (1, 1, 1)$. Perform 5 steps of power iterations to find the largest eigenvalue of A.



And my solution is this.
enter image description here



But, some people says that eigenvalue is 5. This is because when last step, it is based on the middle value. Why?










share|cite|improve this question











$endgroup$












  • $begingroup$
    Remember that is is just an approximation, if you want you can take the average of the ratio of the entries of the two vectors. As you apply the matrix multiple times your values will (possibly, if it has eigenvalues) get better and better.
    $endgroup$
    – Wesley Strik
    Dec 16 '18 at 17:37












  • $begingroup$
    It is not hard for this matrix to determine the actual eigenvalue - although maybe a bit messy. You get a polynomial of the form $(x-4)(x-(4+ sqrt{2}))(x-(4- sqrt{2}))=0$
    $endgroup$
    – Wesley Strik
    Dec 16 '18 at 17:40












  • $begingroup$
    Your method is fine. You assume that $Av = lambda v$ where $v$ an eigenvector, at some point. This assumption need not be always met at all or even after a certain amount of iterations, but, you know, these are approximate methods.
    $endgroup$
    – Wesley Strik
    Dec 17 '18 at 9:26


















1












$begingroup$


$$A = begin{pmatrix}4 &–1 & 0 \-1& 4& -1\ 0 &-1& 4 end{pmatrix} $$ $X_0 = (1, 1, 1)$. Perform 5 steps of power iterations to find the largest eigenvalue of A.



And my solution is this.
enter image description here



But, some people says that eigenvalue is 5. This is because when last step, it is based on the middle value. Why?










share|cite|improve this question











$endgroup$












  • $begingroup$
    Remember that is is just an approximation, if you want you can take the average of the ratio of the entries of the two vectors. As you apply the matrix multiple times your values will (possibly, if it has eigenvalues) get better and better.
    $endgroup$
    – Wesley Strik
    Dec 16 '18 at 17:37












  • $begingroup$
    It is not hard for this matrix to determine the actual eigenvalue - although maybe a bit messy. You get a polynomial of the form $(x-4)(x-(4+ sqrt{2}))(x-(4- sqrt{2}))=0$
    $endgroup$
    – Wesley Strik
    Dec 16 '18 at 17:40












  • $begingroup$
    Your method is fine. You assume that $Av = lambda v$ where $v$ an eigenvector, at some point. This assumption need not be always met at all or even after a certain amount of iterations, but, you know, these are approximate methods.
    $endgroup$
    – Wesley Strik
    Dec 17 '18 at 9:26
















1












1








1





$begingroup$


$$A = begin{pmatrix}4 &–1 & 0 \-1& 4& -1\ 0 &-1& 4 end{pmatrix} $$ $X_0 = (1, 1, 1)$. Perform 5 steps of power iterations to find the largest eigenvalue of A.



And my solution is this.
enter image description here



But, some people says that eigenvalue is 5. This is because when last step, it is based on the middle value. Why?










share|cite|improve this question











$endgroup$




$$A = begin{pmatrix}4 &–1 & 0 \-1& 4& -1\ 0 &-1& 4 end{pmatrix} $$ $X_0 = (1, 1, 1)$. Perform 5 steps of power iterations to find the largest eigenvalue of A.



And my solution is this.
enter image description here



But, some people says that eigenvalue is 5. This is because when last step, it is based on the middle value. Why?







linear-algebra eigenvalues-eigenvectors






share|cite|improve this question















share|cite|improve this question













share|cite|improve this question




share|cite|improve this question








edited Dec 16 '18 at 17:33









Wesley Strik

2,172424




2,172424










asked Dec 16 '18 at 17:19









Genius ParkGenius Park

111




111












  • $begingroup$
    Remember that is is just an approximation, if you want you can take the average of the ratio of the entries of the two vectors. As you apply the matrix multiple times your values will (possibly, if it has eigenvalues) get better and better.
    $endgroup$
    – Wesley Strik
    Dec 16 '18 at 17:37












  • $begingroup$
    It is not hard for this matrix to determine the actual eigenvalue - although maybe a bit messy. You get a polynomial of the form $(x-4)(x-(4+ sqrt{2}))(x-(4- sqrt{2}))=0$
    $endgroup$
    – Wesley Strik
    Dec 16 '18 at 17:40












  • $begingroup$
    Your method is fine. You assume that $Av = lambda v$ where $v$ an eigenvector, at some point. This assumption need not be always met at all or even after a certain amount of iterations, but, you know, these are approximate methods.
    $endgroup$
    – Wesley Strik
    Dec 17 '18 at 9:26




















  • $begingroup$
    Remember that is is just an approximation, if you want you can take the average of the ratio of the entries of the two vectors. As you apply the matrix multiple times your values will (possibly, if it has eigenvalues) get better and better.
    $endgroup$
    – Wesley Strik
    Dec 16 '18 at 17:37












  • $begingroup$
    It is not hard for this matrix to determine the actual eigenvalue - although maybe a bit messy. You get a polynomial of the form $(x-4)(x-(4+ sqrt{2}))(x-(4- sqrt{2}))=0$
    $endgroup$
    – Wesley Strik
    Dec 16 '18 at 17:40












  • $begingroup$
    Your method is fine. You assume that $Av = lambda v$ where $v$ an eigenvector, at some point. This assumption need not be always met at all or even after a certain amount of iterations, but, you know, these are approximate methods.
    $endgroup$
    – Wesley Strik
    Dec 17 '18 at 9:26


















$begingroup$
Remember that is is just an approximation, if you want you can take the average of the ratio of the entries of the two vectors. As you apply the matrix multiple times your values will (possibly, if it has eigenvalues) get better and better.
$endgroup$
– Wesley Strik
Dec 16 '18 at 17:37






$begingroup$
Remember that is is just an approximation, if you want you can take the average of the ratio of the entries of the two vectors. As you apply the matrix multiple times your values will (possibly, if it has eigenvalues) get better and better.
$endgroup$
– Wesley Strik
Dec 16 '18 at 17:37














$begingroup$
It is not hard for this matrix to determine the actual eigenvalue - although maybe a bit messy. You get a polynomial of the form $(x-4)(x-(4+ sqrt{2}))(x-(4- sqrt{2}))=0$
$endgroup$
– Wesley Strik
Dec 16 '18 at 17:40






$begingroup$
It is not hard for this matrix to determine the actual eigenvalue - although maybe a bit messy. You get a polynomial of the form $(x-4)(x-(4+ sqrt{2}))(x-(4- sqrt{2}))=0$
$endgroup$
– Wesley Strik
Dec 16 '18 at 17:40














$begingroup$
Your method is fine. You assume that $Av = lambda v$ where $v$ an eigenvector, at some point. This assumption need not be always met at all or even after a certain amount of iterations, but, you know, these are approximate methods.
$endgroup$
– Wesley Strik
Dec 17 '18 at 9:26






$begingroup$
Your method is fine. You assume that $Av = lambda v$ where $v$ an eigenvector, at some point. This assumption need not be always met at all or even after a certain amount of iterations, but, you know, these are approximate methods.
$endgroup$
– Wesley Strik
Dec 17 '18 at 9:26












1 Answer
1






active

oldest

votes


















0












$begingroup$

You can check with any entry you want, it is indeed approximately $5$ (if you work it out analytically you notice that largest eigenvalue is $4+ sqrt{2}$), but remember that you are using an approximation method, so there is no "correct"way to do it. If you would use more iterations we expect the error to get smaller and to approach the "true" eigenvalue, if it exists of course.





Another, better way to do it is by looking at:
http://college.cengage.com/mathematics/larson/elementary_linear/5e/students/ch08-10/chap_10_3.pdf



Where they show that a good way to do it is by computing the so-called Rayleigh quotient, which if $A^5 x= bar{x} $ is defined to be:



$$ frac{Abar{x} cdotbar{x}}{x cdot x} $$



Give it a try and let us know which $lambda$ you find ?






share|cite|improve this answer











$endgroup$













  • $begingroup$
    when I solve a problem like this, can I get the criteria any row of matrix to find eigenvalue?
    $endgroup$
    – Genius Park
    Dec 17 '18 at 4:51










  • $begingroup$
    I matrix might have imaginary eigenvalues and there is no such thing as a greatest imaginary eigenvalue, because $mathbb{C}$ is not completely ordered. You can find an eigenvalue of largest modulus though. I'm afraid I do not completely grasp your question.
    $endgroup$
    – Wesley Strik
    Dec 17 '18 at 9:21










  • $begingroup$
    If the question is, does it matter which row I pick, no, not really.
    $endgroup$
    – Wesley Strik
    Dec 17 '18 at 9:22










  • $begingroup$
    The assumption is that at some point the vector stabilises and is an eigenvector. Therefore we have that it will be some multiple of itself after applying the matrix. Any entry will be a scalar multiple of the previous entry of the previous vector.
    $endgroup$
    – Wesley Strik
    Dec 17 '18 at 9:24











Your Answer





StackExchange.ifUsing("editor", function () {
return StackExchange.using("mathjaxEditing", function () {
StackExchange.MarkdownEditor.creationCallbacks.add(function (editor, postfix) {
StackExchange.mathjaxEditing.prepareWmdForMathJax(editor, postfix, [["$", "$"], ["\\(","\\)"]]);
});
});
}, "mathjax-editing");

StackExchange.ready(function() {
var channelOptions = {
tags: "".split(" "),
id: "69"
};
initTagRenderer("".split(" "), "".split(" "), channelOptions);

StackExchange.using("externalEditor", function() {
// Have to fire editor after snippets, if snippets enabled
if (StackExchange.settings.snippets.snippetsEnabled) {
StackExchange.using("snippets", function() {
createEditor();
});
}
else {
createEditor();
}
});

function createEditor() {
StackExchange.prepareEditor({
heartbeatType: 'answer',
autoActivateHeartbeat: false,
convertImagesToLinks: true,
noModals: true,
showLowRepImageUploadWarning: true,
reputationToPostImages: 10,
bindNavPrevention: true,
postfix: "",
imageUploader: {
brandingHtml: "Powered by u003ca class="icon-imgur-white" href="https://imgur.com/"u003eu003c/au003e",
contentPolicyHtml: "User contributions licensed under u003ca href="https://creativecommons.org/licenses/by-sa/3.0/"u003ecc by-sa 3.0 with attribution requiredu003c/au003e u003ca href="https://stackoverflow.com/legal/content-policy"u003e(content policy)u003c/au003e",
allowUrls: true
},
noCode: true, onDemand: true,
discardSelector: ".discard-answer"
,immediatelyShowMarkdownHelp:true
});


}
});














draft saved

draft discarded


















StackExchange.ready(
function () {
StackExchange.openid.initPostLogin('.new-post-login', 'https%3a%2f%2fmath.stackexchange.com%2fquestions%2f3042873%2fi-want-to-find-largest-eigenvalue-using-power-iteration-is-it-correct%23new-answer', 'question_page');
}
);

Post as a guest















Required, but never shown

























1 Answer
1






active

oldest

votes








1 Answer
1






active

oldest

votes









active

oldest

votes






active

oldest

votes









0












$begingroup$

You can check with any entry you want, it is indeed approximately $5$ (if you work it out analytically you notice that largest eigenvalue is $4+ sqrt{2}$), but remember that you are using an approximation method, so there is no "correct"way to do it. If you would use more iterations we expect the error to get smaller and to approach the "true" eigenvalue, if it exists of course.





Another, better way to do it is by looking at:
http://college.cengage.com/mathematics/larson/elementary_linear/5e/students/ch08-10/chap_10_3.pdf



Where they show that a good way to do it is by computing the so-called Rayleigh quotient, which if $A^5 x= bar{x} $ is defined to be:



$$ frac{Abar{x} cdotbar{x}}{x cdot x} $$



Give it a try and let us know which $lambda$ you find ?






share|cite|improve this answer











$endgroup$













  • $begingroup$
    when I solve a problem like this, can I get the criteria any row of matrix to find eigenvalue?
    $endgroup$
    – Genius Park
    Dec 17 '18 at 4:51










  • $begingroup$
    I matrix might have imaginary eigenvalues and there is no such thing as a greatest imaginary eigenvalue, because $mathbb{C}$ is not completely ordered. You can find an eigenvalue of largest modulus though. I'm afraid I do not completely grasp your question.
    $endgroup$
    – Wesley Strik
    Dec 17 '18 at 9:21










  • $begingroup$
    If the question is, does it matter which row I pick, no, not really.
    $endgroup$
    – Wesley Strik
    Dec 17 '18 at 9:22










  • $begingroup$
    The assumption is that at some point the vector stabilises and is an eigenvector. Therefore we have that it will be some multiple of itself after applying the matrix. Any entry will be a scalar multiple of the previous entry of the previous vector.
    $endgroup$
    – Wesley Strik
    Dec 17 '18 at 9:24
















0












$begingroup$

You can check with any entry you want, it is indeed approximately $5$ (if you work it out analytically you notice that largest eigenvalue is $4+ sqrt{2}$), but remember that you are using an approximation method, so there is no "correct"way to do it. If you would use more iterations we expect the error to get smaller and to approach the "true" eigenvalue, if it exists of course.





Another, better way to do it is by looking at:
http://college.cengage.com/mathematics/larson/elementary_linear/5e/students/ch08-10/chap_10_3.pdf



Where they show that a good way to do it is by computing the so-called Rayleigh quotient, which if $A^5 x= bar{x} $ is defined to be:



$$ frac{Abar{x} cdotbar{x}}{x cdot x} $$



Give it a try and let us know which $lambda$ you find ?






share|cite|improve this answer











$endgroup$













  • $begingroup$
    when I solve a problem like this, can I get the criteria any row of matrix to find eigenvalue?
    $endgroup$
    – Genius Park
    Dec 17 '18 at 4:51










  • $begingroup$
    I matrix might have imaginary eigenvalues and there is no such thing as a greatest imaginary eigenvalue, because $mathbb{C}$ is not completely ordered. You can find an eigenvalue of largest modulus though. I'm afraid I do not completely grasp your question.
    $endgroup$
    – Wesley Strik
    Dec 17 '18 at 9:21










  • $begingroup$
    If the question is, does it matter which row I pick, no, not really.
    $endgroup$
    – Wesley Strik
    Dec 17 '18 at 9:22










  • $begingroup$
    The assumption is that at some point the vector stabilises and is an eigenvector. Therefore we have that it will be some multiple of itself after applying the matrix. Any entry will be a scalar multiple of the previous entry of the previous vector.
    $endgroup$
    – Wesley Strik
    Dec 17 '18 at 9:24














0












0








0





$begingroup$

You can check with any entry you want, it is indeed approximately $5$ (if you work it out analytically you notice that largest eigenvalue is $4+ sqrt{2}$), but remember that you are using an approximation method, so there is no "correct"way to do it. If you would use more iterations we expect the error to get smaller and to approach the "true" eigenvalue, if it exists of course.





Another, better way to do it is by looking at:
http://college.cengage.com/mathematics/larson/elementary_linear/5e/students/ch08-10/chap_10_3.pdf



Where they show that a good way to do it is by computing the so-called Rayleigh quotient, which if $A^5 x= bar{x} $ is defined to be:



$$ frac{Abar{x} cdotbar{x}}{x cdot x} $$



Give it a try and let us know which $lambda$ you find ?






share|cite|improve this answer











$endgroup$



You can check with any entry you want, it is indeed approximately $5$ (if you work it out analytically you notice that largest eigenvalue is $4+ sqrt{2}$), but remember that you are using an approximation method, so there is no "correct"way to do it. If you would use more iterations we expect the error to get smaller and to approach the "true" eigenvalue, if it exists of course.





Another, better way to do it is by looking at:
http://college.cengage.com/mathematics/larson/elementary_linear/5e/students/ch08-10/chap_10_3.pdf



Where they show that a good way to do it is by computing the so-called Rayleigh quotient, which if $A^5 x= bar{x} $ is defined to be:



$$ frac{Abar{x} cdotbar{x}}{x cdot x} $$



Give it a try and let us know which $lambda$ you find ?







share|cite|improve this answer














share|cite|improve this answer



share|cite|improve this answer








edited Dec 16 '18 at 17:43

























answered Dec 16 '18 at 17:35









Wesley StrikWesley Strik

2,172424




2,172424












  • $begingroup$
    when I solve a problem like this, can I get the criteria any row of matrix to find eigenvalue?
    $endgroup$
    – Genius Park
    Dec 17 '18 at 4:51










  • $begingroup$
    I matrix might have imaginary eigenvalues and there is no such thing as a greatest imaginary eigenvalue, because $mathbb{C}$ is not completely ordered. You can find an eigenvalue of largest modulus though. I'm afraid I do not completely grasp your question.
    $endgroup$
    – Wesley Strik
    Dec 17 '18 at 9:21










  • $begingroup$
    If the question is, does it matter which row I pick, no, not really.
    $endgroup$
    – Wesley Strik
    Dec 17 '18 at 9:22










  • $begingroup$
    The assumption is that at some point the vector stabilises and is an eigenvector. Therefore we have that it will be some multiple of itself after applying the matrix. Any entry will be a scalar multiple of the previous entry of the previous vector.
    $endgroup$
    – Wesley Strik
    Dec 17 '18 at 9:24


















  • $begingroup$
    when I solve a problem like this, can I get the criteria any row of matrix to find eigenvalue?
    $endgroup$
    – Genius Park
    Dec 17 '18 at 4:51










  • $begingroup$
    I matrix might have imaginary eigenvalues and there is no such thing as a greatest imaginary eigenvalue, because $mathbb{C}$ is not completely ordered. You can find an eigenvalue of largest modulus though. I'm afraid I do not completely grasp your question.
    $endgroup$
    – Wesley Strik
    Dec 17 '18 at 9:21










  • $begingroup$
    If the question is, does it matter which row I pick, no, not really.
    $endgroup$
    – Wesley Strik
    Dec 17 '18 at 9:22










  • $begingroup$
    The assumption is that at some point the vector stabilises and is an eigenvector. Therefore we have that it will be some multiple of itself after applying the matrix. Any entry will be a scalar multiple of the previous entry of the previous vector.
    $endgroup$
    – Wesley Strik
    Dec 17 '18 at 9:24
















$begingroup$
when I solve a problem like this, can I get the criteria any row of matrix to find eigenvalue?
$endgroup$
– Genius Park
Dec 17 '18 at 4:51




$begingroup$
when I solve a problem like this, can I get the criteria any row of matrix to find eigenvalue?
$endgroup$
– Genius Park
Dec 17 '18 at 4:51












$begingroup$
I matrix might have imaginary eigenvalues and there is no such thing as a greatest imaginary eigenvalue, because $mathbb{C}$ is not completely ordered. You can find an eigenvalue of largest modulus though. I'm afraid I do not completely grasp your question.
$endgroup$
– Wesley Strik
Dec 17 '18 at 9:21




$begingroup$
I matrix might have imaginary eigenvalues and there is no such thing as a greatest imaginary eigenvalue, because $mathbb{C}$ is not completely ordered. You can find an eigenvalue of largest modulus though. I'm afraid I do not completely grasp your question.
$endgroup$
– Wesley Strik
Dec 17 '18 at 9:21












$begingroup$
If the question is, does it matter which row I pick, no, not really.
$endgroup$
– Wesley Strik
Dec 17 '18 at 9:22




$begingroup$
If the question is, does it matter which row I pick, no, not really.
$endgroup$
– Wesley Strik
Dec 17 '18 at 9:22












$begingroup$
The assumption is that at some point the vector stabilises and is an eigenvector. Therefore we have that it will be some multiple of itself after applying the matrix. Any entry will be a scalar multiple of the previous entry of the previous vector.
$endgroup$
– Wesley Strik
Dec 17 '18 at 9:24




$begingroup$
The assumption is that at some point the vector stabilises and is an eigenvector. Therefore we have that it will be some multiple of itself after applying the matrix. Any entry will be a scalar multiple of the previous entry of the previous vector.
$endgroup$
– Wesley Strik
Dec 17 '18 at 9:24


















draft saved

draft discarded




















































Thanks for contributing an answer to Mathematics Stack Exchange!


  • Please be sure to answer the question. Provide details and share your research!

But avoid



  • Asking for help, clarification, or responding to other answers.

  • Making statements based on opinion; back them up with references or personal experience.


Use MathJax to format equations. MathJax reference.


To learn more, see our tips on writing great answers.




draft saved


draft discarded














StackExchange.ready(
function () {
StackExchange.openid.initPostLogin('.new-post-login', 'https%3a%2f%2fmath.stackexchange.com%2fquestions%2f3042873%2fi-want-to-find-largest-eigenvalue-using-power-iteration-is-it-correct%23new-answer', 'question_page');
}
);

Post as a guest















Required, but never shown





















































Required, but never shown














Required, but never shown












Required, but never shown







Required, but never shown

































Required, but never shown














Required, but never shown












Required, but never shown







Required, but never shown







Popular posts from this blog

Plaza Victoria

In PowerPoint, is there a keyboard shortcut for bulleted / numbered list?

How to put 3 figures in Latex with 2 figures side by side and 1 below these side by side images but in...